Sei sulla pagina 1di 67

Mock Biomed Viva

2015
Compiled by Joshua Loke and edited by Joel Zhou
UNSW Med VI 2015

Page 1 of 67
Table of Contents
Pharmacology.................................................................................................................... 3
Pharmacology 1- DVT...................................................................................................... 3
Pharmacology 2- Inflammatory Bowel Disease...............................................................8
Pharmacology 3- Major depression...............................................................................12
Anatomy.......................................................................................................................... 17
Anatomy 1- Pneumothorax........................................................................................... 17
Anatomy 2- Larynx........................................................................................................ 20
Anatomy 3- Inguinal hernia...........................................................................................23
Pathology......................................................................................................................... 26
Pathology 1- COPD........................................................................................................ 26
Pathology 2- Primary hyperparathyroidism...................................................................31
Pathology 3- Nephrotic syndrome.................................................................................34
Pathology 4- DKA.......................................................................................................... 38
Pathology 5 - Lymphoma...............................................................................................42
Diagnostics...................................................................................................................... 46
Diagnostics 1- Urinary tract infection............................................................................46
Diagnostics 2- meningitis..............................................................................................51
Diagnostics 3- Infective endocarditis............................................................................55

Page 2 of 67
Pharmacology
Pharmacology 1- DVT
A 65-year-old woman presents with unilateral leg pain and swelling of 5 days' duration.
There is a history of hypertension, mild CHF, and recent hospitalisation for pneumonia.
She had been recuperating at home but on beginning to mobilise and walk, the right leg
became painful, tender, and swollen. On examination, the right calf is 4 cm greater in
circumference than the left when measured 10 cm below the tibial tuberosity. Superficial
veins in the leg are more dilated on the right foot and the right leg is slightly redder than
the left. There is some tenderness on palpation in the popliteal fossa behind the knee.

Vital signs
Heart rate (b/min) 80
Blood pressure (mmHg)
128/85
Respiratory rate (/min) 12
Temperature (0C)
36.8

1) What are your provisional and differential diagnoses?


PDx:
o Deep vein thrombosis
DDx:
o Cellulitis
o Lymphedema
o Ruptured popliteal cyst (Baker's cyst)
o Pelvic/thigh mass/tumour compressing venous outflow from the leg
o Compartment syndrome (Trauma)
o Venous insufficiency
Grade Content Remarks
F No PDx or DDx offered
(30m)
P- Mentions only PDx
(50m)
P Mentions PDx and 1 DDx
(70m)
P+ Mentions PDx and 3 DDx
(90m)

2) What investigations will you perform?


Imaging:
Compression US of deep venous system: is first line test in pt with
high/intermediate pretest clinical probability score or elevated D Dimer.
Sensitivity, specificity >95%
- Inability to fully compress lumen of vein using US transducer
- Venous Doppler U/S
- If the patient has a DVT, it will be important to document the extent of the
clot and the degree of luminal occlusion.
Bloods:
D-dimer test fibrin degradation product that is sensitive but not specific for DVT

Page 3 of 67
- Negative result plus a low pretest clinical probability score (as determined
by Wells criteria) excludes DVT
- Positive result can be raised in infection, pregnancy, malignancy, post-op;
do U/S
FBC Leukocytosis if cellulitis. Platelet count (prior to anticoagulation)
UEC prior to anticoagulation
Coags APTT, INR (prior to anticoagulation).
LFTs baseline.
If suspicious of PE
- ABG - low PO2
- ECG tachycardia, RBBB, RV strain (S1Q3T3)
- CXR paucity of vascular markings, peripheral wedge opacity, basal
collapse
- V/Q scan (use with caution in patients with renal impairment)
- CTPA (gold standard) - Use Wells criteria to assess for PE
Thrombophilia tests before commencing anticoagulants if there are no
predisposing factors, recurrent DVT or if there is a family history of DVT

Wells Score for DVT


Active cancer +1
Calf swelling >3cm compared to other side +1
Swollen veins in unilateral superficial veins of affected side (measured 10cm
below tibial tuberosity) +1
Unilateral pitting oedema +1
Entire leg swelling +1
Localised tenderness in deep vein system +1
Paralysis, paresis, cast immobilisation of lower limb +1
Recently bed ridden >3 days or major surgery previous 12 weeks +1
Alternative diagnosis at least as likely -2
Scoring:
o 3 points: high pretest probability treat as suspected DVT and
perform compression US
o 1-2 points: intermediate pretest probability - treat as suspected
DVT and perform compression US
o 0 points: low pretest probability of DVT perform D-dimer test. If +ve,
then treat as suspected DVT and perform compression US. If -ve, DVT
reliably excluded
N.B. the Wells score is usually used to risk stratify patient so doctors dont over use the
D-dimer test

To examiner: If time permits, it is possible to ask candidate to state situations


where V/Q scan is favoured over CTPA:
Allergy to contrast
Pregnancy
CTPA is indeterminate

Grade Content Remarks


F No mention of relevant investigations
(30m)
P- Mentions either D-dimer, CTPA, doppler
(50m) ultrasound
Limited list of investigations

Page 4 of 67
P Mentions Wells score but is not clear what are its
(70m) constituents.
Mentions a good variety of investigations but is
not able to organise them.
Talks about Doppler ultrasound, CTPA
Does not know what are the important
investigations.
P+ Mentions Wells score:
(90m) o Understands what makes up the Wells
score.
o Knows cut offs for further investigation
o Understands the value of the D-dimer
assay when there is a low pretest
probability of DVT
Is able to organise investigations into bedside,
blood and imaging studies, mentioning the most
important tests first.
Is able to state situations where V/Q scan is
favoured over CTPA

3)What pharmacological management will you prescribe for this patient and
what are their mechanisms of action and side effects?
Anticoagulation:
Low molecular weight heparin (LMWH) enoxaparin 1.5mg/kg SC daily or
1mg/kg SC bd
o MOA:
Potentiates effects of antithrombin III inactivation of thrombin
(IIa) & Xa.
Note: inactivates thrombin (IIa) to a lesser extent than UFH because
smaller molecules(2000-6000 vs. 5000-30,000) and therefore
cannot bind to both thrombin and antithrombinsimultaneously.
o S/E:
Heparin induced thrombocytopenic thrombotic syndrome (HITTs)
(ASK MECHANISM of HITTS):
More likely with heparin
Occurs after 5-7 days on heparin with 50% fall from platelet
baseline, cease heparin & administer danaparoid (Orgaran)
Mechanism of HITTS: Heparin binds to platelet factor
4 (PF4) exposes a previously unmasked epitope
increases production of IgG antibody IgG binds to
hepain-PF4 complex forms immune complex that
binds to platelets platelets activated
hypercoagulable state and thrombocytopenia.
Bleeding
Long term side effect- osteoporosis
o Other facts that candidate may mention:
Twice daily dosing preferred for patients at high risk of bleeding or
of thrombus extension e.g. older, obese, malignancy.
Advantages of LMWH over UFH: Longer duration of action
(once/twice daily dosing), response more predictable, no routine lab
monitoring, outpatient management.

Page 5 of 67
Dose adjustment required in the presence of renal impairment
(eGFR<30)
Warfarin:
o Orally, daily (dose according to initiation protocol) then daily dosing
adjusted to an INR of 23
o MOA:
Warfarin inhibits Vitamin K epoxide reductase and quinine
reductase therefore blocking the -carboxylation of glutamic acid
residues therefore production of factors II, VII, IX, X, protein C, S
and Z.
o S/E:
Common (>1%): bleeding
Rare (<0.1%): skin necrosis, purple discolouration of toes, alopecia,
fever, rash, nausea, vomiting, diarrhoea, hepatic dysfunction,
allergic reactions, eg hypersensitivity

Do you know of any other new anticoagulants?


Rivaroxaban:
o Can be used to treat DVT but not recommended in eTG.
o MOA:
Selectively inhibit factor Xa, blocking thrombin production,
conversion of fibrinogen to fibrin, and thrombus development.
o S/E:
Peripheral oedema, itch, skin blisters, muscle spasm
Apixaban
Dabigatran

Grade Content Remarks


F Does not know what medications to give
(30m) No mention of MOA or S/E
P- Mentions LMWH and Warfarin
(50m) Briefly describes the MOA and S/E of the
medications
P Mentions LMWH and Warfarin
(70m) Describes the MOA of these drugs
Mentions S/E of each drug
P+ Mentions LMWH and Warfarin
(90m) Describes the MOA of these drugs
Knows about dose adjustments to heparin in the
setting of renal impairment.
Mentions S/E of each drug
Is able to describe the mechanisms of HITTs
Knows the novel anticoagulants

4) How will you start and monitor warfarin? What is the benefit of LMWH over
UFH?
Starting:
o Exact dosage depends on age and INR. But usually a dose of 5mg would
be given.
o Patients routinely receive doses at 4.00pm during titration protocol, blood
samples for INR obtained 8.00-10.00am (16 hours after previous dose).

Page 6 of 67
o Commence on same day as LMWH: High recurrence of VTE if
commenced alone because warfarin initially prothrombotic
(depletes C&S before factor II, VII, IX, X)
Monitor:
o INR daily until 2 consecutive readings are above INR of 2
o Then weekly for 1 month
o Then 4-6 weeks while stable
Duration of anticoagulation depends on risk of recurrent VTE & bleeding:
o 3 months if DVT provoked by transient major risk factor (eg post-op)
o 3 months if DVT unprovoked, and distal
o 6 months if first unprovoked proximal DVT or PE
o Indefinite if first unprovoked proximal VTE plus active cancer, multiple
thrombophilias, anti-phospholipid antibody syndrome; or recurrent
unprovoked VTE.
Benefit of LMWH over UFH
o Greater bioavailability than unfractionated heparin
o Extensive clinical experience with subcutaneous administration, often
facilitating outpatient treatment
o Longer duration of the anticoagulant effect, permitting administration only
once or twice daily and administration in the outpatient setting
o Better correlation between dose and anticoagulant response, permitting
administration of a fixed dose without laboratory monitoring
o Lower risk of heparin-induced thrombocytopenia
o Lower incidence of osteoporosis

Grade Content Remarks


F Does not know of the need for initial cover with
(30m) heparin
Is not able to state the target INR
Does not know the benefits/differences between
LMWH and UF Heparin
P- Mentions:
(50m) o Initial cover with heparin but does not
understand the reason for doing so
o Target INR of 2-3
Does not know how long the patient needs to be
on warfarin
Understands that there is a difference between
LMWH and UF Heparin
P Understands the need for heparin cover and able
(70m) to provide reasoning
Able to provide target INR of 2-3
Understands some benefit of LMWH over UF
Heparin
P+ Suggests a starting dose for warfarin
(90m) Mentions:
o Initial cover with heparin and reason for
so doing
o Target INR of 2-3
o Knows how often INR should be checked
o Understands that the duration of warfarin

Page 7 of 67
is based on risk of thrombus formation.
Understands all the benefits of LMWH over UF
Heparin

Page 8 of 67
Pharmacology 2- Inflammatory Bowel Disease
A 27-year-old woman presented to the Emergency Department with severe bloody
diarrhoea. The diarrhoea commenced 6 weeks earlier and has gradually worsened. She
was passing up to 6 stools a day containing mucus and blood, and is associated with
crampy abdominal pain. She does not have any similar previous episodes of diarrhoea
but has stated that she had recently returned from Bali about 2 months ago.

On examination, she was pale and dehydrated with generalised tenderness to the left
lower quadrant. Bowel sounds were increased. Digital rectal examination revealed no
masses but dark red blood and mucus were present on the glove.

Vital signs
Heart rate (b/min)
112
Blood pressure (mmHg)
110/70
Respiratory rate (/min) 18
Temperature (0C)
37.8

1) What are your provisional and differential diagnoses?


PDx: Inflammatory Bowel Disease namely Ulcerative colitis or Crohns
disease
DDx:
o Gastroenteritis:
Viral: norovirus, rotavirus (but no blood)
Bacteria: Salmonella, Shigella, Campylobacter, enteroinvasive e.
coli
Parasitic/other: giardia, entamoeba
o Coeliac disease
o Medications (pseudomembranous colitis, laxatives)
Other considerations, particularly when older:
o Ischaemic colitis
o Malignancy
o Diverticulitis
Grade Content Remarks
F No PDx or DDx offered
(30m)
P- Mentions only PDx
(50m)
P Mentions PDx and 1 DDx
(70m)
P+ Mentions PDx and 3 DDx
(90m) Demonstrates ability to structure DDx

2) What investigations will you do?


Bedside:
Stool sample:
o MCS: exclude infections
o OPS (Ova, Parasites, Spores): exclude parasitic infections

Page 9 of 67
o C. difficile toxin test: exclude pseudomembranous colitis if recent antibiotic
use
Blood tests:
FBC:
o Anaemia (IBD chronic disease OR blood loss OR malabsorption of iron,
folate, B12)
CRP & ESR:
o in inflammation
EUC:
o Na& K from diarrhoea (unlikely to have metabolic acidosis from
hypoperfusion)
LFT:
o ALP & bilirubin from Primary Sclerosing Cholangitis (associated with UC),
baseline
Albumin:
o (chronic diarrhoea/malabsorption)
Blood cultures:
o -ve, exclude bacteraemia and infectious diarrhoea when this sick + fever
Serologic testing for IBD:
o IgA and IgG antibodies to anti-Saccharomyces cerevisiae (ASCA) for
Crohns disease
o p-ANCA for UC
TFTs: TSH &T4 = hyperthyroidism (if diarrhoea was not bloody)
Test for Coeliac disease:
o IgA
o Anti- endomysial
o Anti-transglutaminase
o Anti-endomysial antibody
Imaging:
CXR: exclude perforation
AXR: UC no faecal shadows, colonic dilation, mucosal thickening. Skip lesions
(think Crohns). Ensure not toxic megacolon >6cm (risk of perforation)
Barium Enema (not during attack):
o UC: loss of haustra, granular mucosa, shortened colon
o Crohns: cobblestoning, rose thorn ulcers, colon strictures with rectal
sparing
Colonoscopy (not if toxic megacolon perforation risk)

Grade Content Remarks


F No mention of relevant investigations
(30m)
P- Mentions non-appropriate investigations
(50m)
P Mentions a good variety of investigations but is
(70m) not able to organise them.
Provides the rationale for the proposed
investigations.
P+ Items in P and:
(90m) Is able to organise investigations into bedside,
blood and imaging studies, mentioning the most
important tests first.
Mentions tests that will help differentiate Crohns
from UC:

Page 10 of 67
o IgA and IgG antibodies to anti-
Saccharomyces cerevisiae (ASCA)
o p-ANCA

3) What are some complications associated with ulcerative colitis?


Intracolonic Sx remember "COLITIS"
Colon Adenocarcinoma
o Original data suggested high risk of progression to cancer in UC patients
o Previously, most patients were advised to have total colectomy
o Risk of colon cancer may be in UC patients with UC taking medications
o Risk in UC patients after 10 years of disease
o Prior to aggressive treatment, colon cancer risk was >40% at 24 years
o Frequent colonoscopy with biopsies is recommended for surveillance
Obstruction
Leakage - perforation
Iron deficiency - hemorrhage
Toxic Megacolon (see below)
Inanition- exhaustion caused by lack of nourishment.
Strictures, Fistulas (rare in UC), perirectal abscess

Extracolonic Sx- remember "ULCERATIVE"


Urinary Calculi - oxalate stones
Liver Disease: Sclerosing Cholangitis
Cholelithiasis
Erythema nodosum (pyoderma gangrenosum)
Retardation of growth
Arthritis
o Proximal Arthritis associated with IBD most common in HLA-B27+ patients
o Distal polyarthritis also seen in IBD
o Incidence of true ankylosing spondylitis is elevated in IBD (B27+)
Thrombophlebitis- portal vein
Iatrogenic: glucocorticoids, blood transfusions, operations
Vitamin Deficiency (Malabsorption)
Eyes: uveitis, chorioretinitis, iridocyclitis, episcleritis

Grade Content Remarks


F No mention of relevant complications
(30m)
P- Mentions 4 complications
(50m)
P Mentions 5 complications
(70m)
P+ Mentions 5 complications
(90m) Is able to classify complications into Intra- and
extracolonic complications

4) She is being treated with prednisolone and sulfasalazine. What other


medications are there? What are the mechanisms of action of these drugs at
the molecular level? What are their side effects?
Would you use azathioprine?

Drug Mechanism of action S/E


Prednisone Enters cell binds to GR and GR Steroid side effects

Page 11 of 67
(corticoster steroid-receptor complex enters CUSHINGOID:
oid) nucleus andeffects gene Cataracts
transcription annexin-1 (protein) Ulcers
(& COX-2 directly) Skin: striae, thinning,
phospholipaseA2 COX2 bruising
PG, LT, PAF Hypertension/ Hirsutism/
Hyperglycemia
Infections
Necrosis, avascular necrosis
of the femoral head
Glycosuria
Osteoporosis, obesity
Immunosuppression
Diabetes
Sulfasalasin A combination of 5-aminosalicylic Vomiting, Oligospermia, Mild
e acid (an anti-inflammatory) and haemolysis
sulfapyridine (an antibacterial).
Toxicity to lymphocytes > other
cells and immunoregulation
reduced T cell mediated
inflammation.
Mesalazine 5-aminosalicylic acid (5-ASA) which Common (>1%): nausea,
has an anti-inflammatory action on rash, headache, diarrhoea
colonic mucosa. Infrequent (0.11%):
Toxicity to lymphocytes > other interstitial nephritis
cells and immunoregulation Rare (<0.1%): blood
reduced T cell mediated dyscrasias, pancreatitis
inflammation. (reversible), hepatitis
Azathioprin Metabolised to mercaptopurine Myelosuppression, Alopecia,
e altered purine synthesis impairs Diarrhoea, Mouth ulcers,
cellularimmunity, cell Oesophagitis
proliferation, inhibited inflammatory
response
Methotrexat Competitive dihydrofolate Nausea, Mouth ulcers,
e (+ Folic reductase inhibitor AST/ALT (esp with
Acid) Inhibit DNA synthesis and cell leflunomide),
replication by inhibiting folic acid Myelosuppression,
conversion cytotoxic, Pneumonitis/fibrosis. NB:
immunosuppressive and anti- SE with folic acid
inflammatory action supplementation
FOR UNRESPONSIVE UC (Cyclosporin and Infliximab)
Cyclosporin Calcineurin inhibitor Gingival hyperplasia &
Cyclosporin forms complexes with hirsutism
cyclophilin which then blocks the
action of calcineurin in activated T
cells preventproduction of IL-2 &
other cytokines T cell
stimulation & proliferation
Infliximab TNF antagonist Delayed hypersensitivity,
Binds to TNF alpha (cytokine from Serum sickness (Type 3
macrophages for inflammatory & hypersensitivity reaction to
immune responses) foreign protein/serum)

Page 12 of 67
Grade Content Remarks
F Is not able to state other possible drugs used
(30m) Unclear about the mechanism of action and S/E
of the medications (e.g. N/V/D)
P- Is able to list 2 other drugs
(50m) Briefly describes their mechanisms of action
Unclear about the S/E of these drugs
P Is able to >2 other drugs
(70m) Describes in detail their mechanisms of action
and S/E
P+ Is able to >2 other drugs
(90m) Describes in detail their mechanisms of action
and S/E
Is able to state the indications of the drugs used

Page 13 of 67
Pharmacology 3- Major depression
A 32 year-old single mother with 2 children under age 5 comes to see you, her GP,
complaining about headaches and lack of energy. She is unemployed and divorced 6
months ago. On further questioning, she reveals that she has had little appetite and has
lost 5kg in the last two months. She struggles to fall asleep, often wakes up in the middle
of the night and finds it hard to get back to sleep. She has lost interest in doing things
that she used to enjoy, like walking and gardening, as well as worrying about her
parenting. She finds herself losing her temper easily and yells at her children, leading
her to believe that she is a bad mother. She often feels like crying for no apparent
reason.

On discussing the situation with this woman, you believe that she is suffering from major
depression. You discuss options for therapy, including CBT and pharmacotherapy. She
does not feel she can commit to CBT at the present time.

1) What features of depression does this woman exhibit? How severe is her
depression? What other differential diagnoses would you consider?

N.B.: criteria for major depression: according to DSM-V:


5 depressive symptoms (SIG E CAPS- Suicidal thoughts, Interest , Guilt,
Energy , Conc. , Appetite or , Psychomotor changes (agitation or slowing),
sleep or ) and
Depressed mood or anhedonia
For at least 2 weeks

This patient has:


energy
appetite
sleep
Anhedonia
Depressed mood
Been having symptoms for 6 months
She has major depression (5 symptoms)

DDx:
Organic causes:
Alcohol abuse
Illicit drug use
Hypothyroidism
Vit B12/Folate deficiency
Addisons disease
Medication (Antihypertensives [beta blockers; methyldopa]; steroids; H2 blockers
[ranitidine]; benzoz; muscle relaxants; appetite suppressants; chemotherapy
agents)
Infections (Lyme disease; Infective mononucleosis; HIV encephalopathy; Syphillis)
Inflammatory conditions (SLE)

Other pyschiatric conditions:


Situational adjustment reaction with depressed mood
Bipolar disorder

Page 14 of 67
Premenstrual dysphoric disorder (PMDD)
Grief reaction
Anxiety disorders

Grade Content Remarks


F Is not able to identify symptoms of depression in
(30m) the stem
P- Demonstrates ability to identify symptoms of
(50m) depression in the stem
P Demonstrates ability to identify symptoms of
(70m) depression in the stem
Mentions PDx (Major depression) and 1 DDx
P+ Demonstrates ability to identify symptoms of
(90m) depression in the stem
Identifies that this patient has major depression
Mentions PDx and 3 DDx
Can do the abovementioned without much
assistance from the examiner

2) What classes of anti-depressant drugs are available, and which would you
prescribe in this case? Justify your answer.
Antidepressant drugs:
The commonest two pharmacological types of anti-depressants are:
Selective serotonin reuptake inhibitors (SSRIs)
Tricyclic antidepressants (TCAs)
Others:
Serotonin-noradrenaline (norepinephrine) reuptake inhibitor (SNRI)
5-HT2 receptor antagonist (mirtazapine)
Bupropion (dopamine re-uptake inhibitor)- NOT used in Australia.
Which would I prescribe?
The choice of medication should be based on:
Patient preference, tolerability
Past evidence of effectiveness in the patient or in a family member (but
this is a NEW patient).
According to the evidence, the best therapy for this patient is:
Psychotic Major depression Minor depression
depression
1st line -ECT Antidepressant + Antidepressant or
-Antidepressant + psychotherapy psychotherapy
psychotherapy
If 1st -ECT Change antidepressant Change antidepressant
line -Antidepressant + regimen: regimen:
fails psychotherapy 1) TCA 1) TCA
+ 2) SSRI/SNRI + mirtazapine 2) SSRI/SNRI +
-Antipsychotic mirtazapine
3) ? Adding atypical
antipsychotic

Grade Content Remarks


F Does not mention SSRIs
(30m)
P- Mentions SSRI

Page 15 of 67
(50m)
P Mentions SSRI + TCA + SNRIs and mirtazapine
(70m)
P+ Mentions SSRI + TCA + SNRIs and mirtazapine
(90m) Knows that SSRIs are the most commonly used
antidepressant
Takes into account patient preference, tolerability
and previous evidence of

3) Describe the mechanism of action and side effects of SSRIs and other drugs
that can be used to treat depression.
Dru Mechanism of Example S/E Comments
g action s
SSRI Inhibit the reuptake Citalopra -S/E resemble a hangover- -Cause less
of the monoamine m N+V, headache, diarrhea and S/E compared
serotonin (5-HT) Escitalopr dry mouth. to tricyclics
within the synapse. am -Insomnia and paradoxical e.g. SSRIs do
Fluvoxami agitation can occur when not cause sig.
ne first starting the drugs. weight-gain
Fluoxetine -SEROTONIN syndrome when -Long half-life
Paroxetine ingesting 2 drugs e.g. (possible to
Sertraline SSRI+ MAOI or dopaminergic give OD). Easy
drugs (e.g. selegiline) or a to comply.
TCA. Therefore
-SEROTONIN syndrome SSRIs are first-
(remember HARMED): line treatment.
Hyperthermia
Autonomic instability
Rigidity
Myoclonus
Encephalopathy
Diaphoresis

TCA They potentiate the Dosulepin In long-term treatment or Because of


action of the Imipramin prophylaxis, weight gain is their toxicity in
monoamines, e most troublesome. overdose do
noradrenaline Amitriptyli 1) Antimuscarinic effects NOT prescribe
(norepinephrine) and ne Dry mouth, constipation, them to
serotonin, by tremor, blurred vision, potentially
inhibiting their urinary retention suicidal
reuptake into nerve 2) Cardiovascular outpatients
terminals QT prolongation,
arrhythmias, postural
hypotension
3) Convulsant activity
seizure threshold
Other effects
Weight gain, Sedation, Mania
(rarely)

SNR Venlafaxine: Venlafaxin Venlafaxine:

Page 16 of 67
Is -SNRI. e -Nausea
Mirtazapine Mirtazapin -Hypertension at high doses
-5-HT2 and 5-HT3 e Mirtazapine:
receptor antagonist Raboxetin -Sedating at low doses
-a potent 2- e -Weight gain
adrenergic blocker. -Agranulocytosis (rare)
( NA and selective Raboxetine
serotonin -Dry mouth
transmission). -Insomnia
Reboxetine -Constipation
-Selective -Urinary hesitancy
noradrenaline -Tachycardia
reuptake inhibitor.
MAO Irreversibly Phenelzin - appetite MAOIs should
I inhibiting the e -Weight gain not be given
intracellular -Erectile dysfunction within 2 weeks
enzymes -Insomnia of a SSRI
monoamine oxidase
A and B, of
noradrenaline,
dopamine and 5-
hydroxytryptamine
in the brain

Grade Content Remarks


F Is not able to state other possible drugs used
(30m) Unclear about the mechanism of action and S/E
of the medications (e.g. N/V/D)
P- Is able to list 2 other drugs
(50m) Briefly describes their mechanisms of action
Unclear about the S/E of these drugs
P Is able to >1 other drugs
(70m) Describes in detail their mechanisms of action
and S/E
P+ Is able to >2 other drugs
(90m) Describes in detail their mechanisms of action
and S/E

Page 17 of 67
4) What information would you give this woman regarding potential drug-drug
interactions?
SSRI:
SEROTONIN syndrome when ingesting 2 drugs e.g. SSRI+ MAOI or
dopaminergic drugs (e.g. selegiline) or a TCA.
SEROTONIN syndrome (remember HARMED):
Hyperthermia
Autonomic instability
Rigidity
Myoclonus
Encephalopathy
Diaphoresis

MAOI:
o Produce a severe and dangerous hypertensive reaction with foods
containing tyramine or dopamine and therefore a restricted diet is
prescribed.
o Tyramine: present in cheese, pickled herrings, yeast extracts, certain red
wines, and any food, such as game, that has undergone partial
decomposition.
o Dopamine: present in broad beans.
o MAOIs interact with drugs such as pethidine and can also occasionally
cause liver damage.
o MAOIs should not normally be given within 2 weeks of a serotonin reuptake
inhibitor, depending on half-lives.

Grade Content Remarks


F Is not able to answer the question even after
(30m) prompting
P- Know about the drugs interactions that may bring
(50m) about the serotonin syndrome
Is not able to state what signs and symptoms one
can expect with the serotonin syndrome
P Know about the drugs interactions with SSRIs
(70m) that may bring about the serotonin syndrome
Knows about signs and symptoms of serotonin
syndrome
P+ Know about the drugs interactions with SSRIs
(90m) that may bring about the serotonin syndrome
Knows about signs and symptoms of serotonin
syndrome
Is able to identify that tyramine, dopamine and
pethidine can interact with MAOI

Page 18 of 67
Anatomy
Anatomy 1- Pneumothorax
A 20-year-old man was brought in by ambulance to the emergency department after a
motor vehicle accident with complaints of right-sided chest pain and shortness of breath.
He states that these symptoms began suddenly. He has no significant past medical
history but has smoked cigarettes since the age of 16 years.
The following are his vital signs:

Vital signs
Heart rate (b/min)
140
Blood pressure (mmHg)
90/65
Respiratory rate (/min) 24
Temperature (0C)
36.8

1) A Pan Scan was performed. Describe the following CT scan, name the
labelled structures and from the scan, determine the cause for this mans
shortness of breath.

Page 19 of 67
F
A

C E

This is an axial CT scan taken at T5-6with lung windows. The labelled structures
are:
o A- Ascending aorta
o B- Descending aorta
o C- Left pulmonary artery
o D- Superior vena cava
o E- Pulmonary trunk
o F- Right ventricle
o G- Azygos vein

Grade Content Remarks


F Is not able to identify that this is a pneumothorax
(30m) Identifies at least 3 structures
P- Mentions pneumothorax as the diagnosis
(50m) Identifies at least 4 structures
P Mentions pneumothorax as the diagnosis
(70m) Identifies at least 5 structures
P+ Mentions pneumothorax as the diagnosis
(90m) Identifies all 7 structures

Page 20 of 67
Mentions that he/she is given an axial CT scan
with lung windows
Is able to identify the approximate level in which
the CT scan is taken

2) What abnormalities are present in the CT?


Fractured ribs (Flail segment but difficult to tell on this view)
Subcutaneous emphysema
Haemothorax / Pleural effusion
Chest drain
Atelectasis of lung
Easily identifiable visceral pleural line
Hyperexpansion of ipsilateral hemithorax
Compression/ Collapsed great veins

Grade Content Remarks


F Offers no answers
(30m)
P- Mentions 2 abnormalities
(50m)
P Mentions 3 abnormalities
(70m)
P+ Mentions 4 abnormalities
(90m)

3) What would you expect to find on examination?


Dyspnoea at rest.
Patient will be in shock (Hypotension and tachycardia)
The affected hemithorax will be hyperexpanded with decreased chest excursion.
The affected hemithorax is hyper-resonant on percussion, with diminished breath
sounds when compared with the normal hemithorax.
His cardiovascular examination is usually normal.
The trachea will be shifted to contralateral side- associated with tension
pneumothorax.
Distended neck veins

Grade Content Remarks


F Offers no answers
(30m)
P- Mentions 2 abnormalities O/E
(50m)
P Mentions 3 abnormalities O/E
(70m)
P+ Mentions 4 abnormalities O/E
(90m)

4) Can you describe the mechanism of a tension pneumothorax?


Pneumothorax: to gas within the pleural space.
Normally, alveolar pressure > intrapleural pressure, while intrapleural pressure <
atmospheric pressure.

Page 21 of 67
Therefore, if a communication develops between an alveolus and the pleural
space or between the atmosphere and the pleural space, gases will follow the
pressure gradient and flow into the pleural space.
This flow will continue until the pressure gradient no longer exists or the abnormal
communication has been sealed.
Because the thoracic cavity is normally below its resting volume, and the lung is
above its resting volume, the thoracic cavity enlarges and the lung becomes
smaller when a pneumothorax develops.
A tension pneumothorax is a medical emergency and occurs when the
intrapleural pressure exceeds atmospheric pressure, especially during
expiration, and results from a ball valve mechanism that promotes inspiratory
accumulation of pleural gases.
The build-up of pressure within the pleural space eventually results in
hypoxaemia and respiratory failure from compression of the lung.

Grade Content Remarks


F Offers no answers
(30m)
P- Unclear about the mechanism of tension PTX-
(50m) mentions some key words, but is not able to
systemically explain the pathophysiology
underpinning the presentation
P Describes the mechanism of tension PTX only
(70m) briefly
P+ Describes the mechanism of tension PTX in great
(90m) detail
Mentions differences between alveolar,
intrapleural, intrapleural and atmospheric
pressures leading to the presentation

5) How do you insert a chest tube? (Extra question that the examiners might
ask)
Triangle of safety (Mid axillary line; Border of pectoralis major; 5 th intercostal
space)
Drape and prep the area as appropriate
Use local anaesthetic (1% Lignocaine), inject over the area
Incision is made above the rib to avoid the neurovascular bundle
Blunt dissection using finger until the pleural surface is reached
Make an incision and use finger to feel around the area to ensure in right direction
Insert drain with a clamp
Connect to underwater seal
Suture

Page 22 of 67
Anatomy 2- Larynx
A 57-year-old man presents with a 6-month history of hoarseness. Over the past week he
has noted progressive difficulty breathing. He also has otalgia, dysphagia, odynophagia
(painful swallowing), and a 9-kg weight loss. He has an 80-pack-year tobacco history and
drinks 8 beers per day. Neck examination demonstrates a right-sided mass that is firm
and fixed. There is mild biphasic stridor with deep inspiration and expiration, but the
patient has no increased work of breathing at rest, and breath sounds are clear.

1) What is your provisional diagnosis for this patient and other differential
diagnoses for hoarseness?
PDx: Laryngeal cancer (must rule out, especially if >2 weeks)
DDx:
o Infection:
Laryngitis
Epiglottitis
o Inflammation:
Toxic
Fumes
Reflux
Smoking
o Functional
o Overuse:
Singers nodules
o Neurological
Central strokes, tumours, MS
Peripheral laryngeal nerves
o Infiltrative causes:
Sarcoidosis
o Autoimmune:
Wegener's granulomatosis: Patients may present with nasal
congestion, nasal crusting, cough and difficulty breathing, and
haematuria.

Grade Content Remarks


F No PDx or DDx offered
(30m)
P- Mentions only PDx
(50m)
P Mentions PDx and 2 DDx
(70m)
P+ Mentions PDx and 4 DDx
(90m) Is able to categorise DDx

2) Identify the following structures and identify the anatomical levels of


structures A, E and F:
State the approximate levels of B, E and F.
A- epiglottis

Page 23 of 67
B- hypoid cartilage (C2)
C- corniculate cartilage
D- arytenoid cartilage
E- thyroid cartilage (C4)
F- cricoid cartilage (C6, usually at the
sternal notch)
G- trachea
H- false cord
I- ventricle of morgani or laryngeal
ventricle
J- vocal cord

Grade Content Remarks


F Identifies 5 structures
(30m)
P- Identifies at least 6-7structures
(50m)
P Identifies at least 8-9 structures
(70m)
P+ Identifies all 10 structures
(90m) Knows the approximate levels of B, E and F

3) Can you describe the intrinsic musculature of the larynx and their effects on
the vocal ligaments?
Abduction:posterior crico-arytenoid muscles
Adduction:Transverse and oblique arytenoid muscles
Slackening: vocalis and thyroarytenoid muscles
Tensing: cricothyroid muscles

Page 24 of 67
Grade Content Remarks
F Is not able to answer the question
(30m)
P- Mentions 2 muscles
(50m)
P Mentions 3-5 muscles
(70m)
P+ Mentions all 6 structures
(90m) Is able to express answer in a manner that
demonstrates understanding of the function of
each muscle

4) Describe the neurological pathways which allow phonation.


Recurrent laryngeal nerves:
o From the vagus nerves in the inferior part of the neck.
o Right recurrent laryngeal nerve loops below the right subclavian
artery.
o Left recurrent laryngeal nerve loops under the arch of the aorta.

Page 25 of 67
o They pass posteromedially to the thyroid along the tracheooesophageal
groove to supply all the muscles of the larynx except the cricothyroid
Cricothyroid muscle is the only laryngeal muscle supplied by the branch of the
vagus nerve known as the EXTERNAL BRANCH OF THE SUPERIOR
LARYNGEAL NERVE (rather than the recurrent laryngeal nerve).

Grade Content Remarks


F Is not able to answer the question
(30m)
P- Is aware of the recurrent laryngeal nerves but is
(50m) unclear about its pathway to the larynx
P Is aware of the recurrent laryngeal nerves
(70m) Describes in detail their pathways to the larynx
P+ Is aware of the recurrent laryngeal nerves
(90m) Describes in detail their pathways to the larynx
Understands that the cricothyroid muscleis
separately innervated by the EXTERNAL BRANCH
OF THE SUPERIOR LARYNGEAL NERVE

5) Describe the epithelium covering the larynx


Other than false vocal cords: respiratory epithelium- pseudostratified ciliated
columnar epithelium with goblet cells
False vocal cords: stratified non-keratinised squamous epithelium

6) What investigations will be useful in this man?


Imaging:
flexible fibre-optic laryngoscopy: lesion involving the vocal folds, epiglottis,
arytenoids, or aryepiglottic folds
Neck CT with contrast: laryngeal mass may show cartilage invasion; presence or
absence of cervical lymphadenopathy
Chest CT with contrast: metastatic nodules, lymphadenopathy
Whole-body PET/CT scan: uptake in areas of malignancy and metastasis
Histopathology:
FNA of neck mass: squamous cell carcinoma

Grade Content Remarks


F Is not able to answer the question
(30m)
P- Offers 1-2 investigations
(50m)
P Mentions 3-4 investigations and gives reasons for
(70m) doing the investigations
P+ Mentions 3-4 investigations and gives reasons for
(90m) doing the investigations
Is able to categorise the investigations

Anatomy 3- Inguinal hernia


A 68-year-old retired labourer presents to his primary medical doctor with a 3-week
history of a dull dragging discomfort in his right groin toward the end of the day. The
discomfort is associated with a lump while standing but disappears when lying supine.

Page 26 of 67
He denies any other significant past medical or surgical history. On physical examination,
a bulge is present when standing that disappears when supine.

1) What is your provisional diagnosis and differential diagnoses?


PDx: Inguinal hernia
DDx:
o Hernia:
Femoral hernia:
Femoral hernia is located below the inguinal ligament
Lateral and inferior to the pubic tubercle
o Testicular:
Undescended testis:
Inguinal hernia is associated with fully developed
hemiscrotum and the bulge in the groin is associated with
expansible cough impulse.
Hydrocele
Hydrocele of the spermatic cord
Lipoma of the spermatic cord
Spermatocele
o Vascular:
Femoral aneurysm:
Pulsatile mass in the groin. Inguinal hernia should not pulsate.
Saphena varix
o Renal:
Transplanted kidney
o Other:
Lymphadenopathy
Sebaceous cyst
Psoas abscess:
May present with back pain and fever. Recent history of
foreign travel or contact with someone with chronic cough,
weight loss, and night sweats are not uncommon.
Grade Content Remarks
F Is not able to state any DDx
(30m)
P- Unable to provide a PDx but is able to offer 1-2
(50m) DDx
P Mentions PDx and 3 DDx
(70m)
P+ Mentions PDx and 4 DDx
(90m)

2) What types of inguinal hernias do you know? How would you differentiate
them(based on anatomical landmarks)?
Direct:
o Hernia sac comes through the inguinal floor medial to the inferior
epigastric artery and the deep inguinal ring.
o Comes through Hesselbach's triangle:
Medial border- Lateral margin of the rectus sheath, also called linea
semilunaris
Superolateral border- inferior epigastric vessels
Inferior border- inguinal ligament (Poupart's ligament)

Page 27 of 67
Indirect:
o Hernia sac comes through the internal (deep) inguinal ring, lateral to the
epigastric artery.
Pantaloon hernia: ipsilateral, concurrent direct + indirect inguinal hernias.
Sliding hernia:
o Important subclassification of indirect hernia
o Bowel fused to the peritoneum comes through the internal inguinal ring.
Strangulated:
o Blood supply of the hernia contents is compromised ischaemia
gangrene and perforation of the affected bowel segment.
o A strangulated hernia can also contain omentum or other viscera, such as
bladder.

Grade Content Remarks


F Unable to answer
(30m)
P- Mentions only direct and indirect inguinal hernias
(50m) Is unsure of the anatomical distinctions
P Mentions only direct and indirect inguinal hernias
(70m) Is able to describe the anatomical distinctions of
indirect and direct hernias
P+ Is able to describe the anatomical distinctions of
(90m) indirect and direct hernias
Mentions other types of inguinal hernias (e.g.
pantaloons, strangulated, sliding hernias)

3) Describe the anatomy of the inguinal canal.


Internal inguinal ring: lie in the mid-point of the inguinal ligament- between ASIS
and PT
The femoral artery comes out from the mid-inguinal point- between ASIS and
pubic symphysis
Inguinal canal anatomy (Inguinal Canal Walls):
o 2 xMALT 2M, 2A, 2L, 2T; Corresponds to Superior, Anterior,
Inferior and Posterior walls.
2 Muscles (Superiorly) = Internal Oblique/transverses Abdominis
2 Aponeuroses (Anteriorly) = External Oblique Apo/ Internal Oblique
Apo.
2 Ligaments (Inferiorly) = Inguinal/ Lacunar
2 Ts (Posteriorly) = Transversalis fascia, conjoint Tendon.

Page 28 of 67
Grade Content Remarks
F Unable to answer
(30m)
P- Mentions 2-3 anatomical structures of the
(50m) inguinal canal
P Mentions 4-6 anatomical structures of the
(70m) inguinal canal
P+ Mentions 8 anatomical structures of the
(90m) inguinal canal

4) What are the contents of the spermatic cord? What is the purpose of the
pampiniform plexus?What are the layers of the testes?
Contents of Spermatic Cord:
3 Arteries:
o 1. Testicular
o 2. Cremasteric
o 3. Artery of Ductus Deferens
3 other vessels:
o 1. Pampiniform (L: vines wrapping) plexus
o 2. Cremasteric vein
o 3. Lymphatics
3 Nerves:
o 1. Genital branch of genitofemoral nerve (NB: ilioinguinal n is in the canal
not the cord)
o 2. Testicular sympathetic nerve
o 3. Autonomic plexus of ductus deferens
Grade Content Remarks
F Unable to answer
(30m)
P- Mentions 2-3 constituents of the spermatic cord
(50m) Mentions 2-3 layers of the testes
P Mentions 4-6 constituents of the spermatic cord
(70m) Mentions 4-6 layers of the testes
P+ Mentions 7 constituents of the spermatic cord
(90m) Mentions all the layers of the testes
Understands the purpose of the pampiniform
plexus

Page 29 of 67
Pathology
Pathology 1- COPD

1) ASK FOR WHITE BOOK (spend at most2mins on this segment): Test


examined: ______________________
Select one anatomical cut up the candidate observed. Please discuss the
pathological processes that contributed to the specimen you saw. OR
Select a morgue case. Have the candidate to describe the process of dissecting
the corpse.
Grade Content Remarks
F Has not prepared for the case at all
(30m)
P- Poorly describes the anatomical cut up or morgue
(50m) case
P For morgue:
(70m) o Detailed step-by-step explanation of how
the process of cutting up the patient
For anatomical cut up:
o Macroscopic and/or microscopic findings
P+ Opens with the history of the patient and how the
(90m) specimen was obtained
States the indication for the anatomical or
morgue cut up
For morgue:
o Detailed step-by-step explanation of how
the process of cutting up the patient
o Is able to relate these findings to the
pathology underpinning the disease
process
For anatomical cut up:
o Macroscopic and microscopic findings
o Is able to relate these findings to the
pathology underpinning the disease
process

Invite candidate to read the case history


A 66-year-old man with a smoking history of one pack per day for the past 47 years
presents with progressive shortness of breath and chronic cough, productive of yellowish
sputum, for the past 2 years. On examination he appears cachectic and in moderate
respiratory distress. His neck veins are mildly distended. Lung examination reveals a
barrel chest and poor air entry bilaterally, with moderate inspiratory and expiratory
wheezing. Heart and abdominal examination are within normal limits. Lower extremities
exhibit scant pitting oedema. Within hours of his admission, the nursing staff report that
he has become drowsy and difficult to rouse.

Vital signs
Heart rate (b/min)
106
Blood pressure (mmHg)

Page 30 of 67
140/70
Respiratory rate (/min) 26
Temperature (0C)
37.5

1) What investigations will be important in this patient?


Gold standard:
Lung function tests (3 components)
1. Spirometry
2. Lung Volume
3. Diffusion Capacity
Bedside:
Spirometry: obstructive picture- FEV1/FVC ratio <70%; total absence of
reversibility is neither required nor the most typical result
Pulse oximetry: low oxygen saturation. An oxygen saturation of 88-90% may be
acceptable.
ECG: signs of right ventricular hypertrophy, arrhythmia, ischaemia
Sputum culture: Presence of purulent sputum is sufficient to commence empirical
antibiotics. Sputum culture indicated if empirical antibiotics fail.
Blood tests:
ABG (no VBG please!): PaCO2>50 mmHg and/or PaO2 of <60 mmHg suggests
respiratory insufficiency
FBC: haematocrit, possible WCC
Imaging studies:
CXR: hyperinflation
Grade Content Remarks
F No mention of relevant investigations
(30m)
P- Mentions non-appropriate investigations
(50m)
P Mentions a good variety of investigations but is
(70m) not able to organise them.
MUST mention ABG, spirometry and CXR
Provides the rationale for the proposed
investigations.
P+ Items in P and:
(90m) Is able to organise investigations into bedside,
blood and imaging studies, mentioning the most
important tests first.

2) Can you describe the following ABG results of this patient?


N.B.: Once the candidate mentions ABG, immediately show them the ABG results:
ABG
pH: 7.15 7.35-7.45
PO2: 52 80-100
PCO2: 60 35-45
HCO3: 32 24-34
Base access: +6 -
3<BE<3

Answer: chronic respiratory acidosis with metabolic compensation.


See below for ABCs of ABG (worth highlighting this in the feedback session).

Page 31 of 67
Grade Content Remarks
F Unable to read ABG at all
(30m)
P- Is able to deduce that this is a respiratory
(50m) acidosis
Unable to determine the nature of compensation
P Is able to deduce that this is a respiratory
(70m) acidosis
Is able to determine that there is adequate
metabolic compensation
P+ Is able to deduce that this is a chronic
(90m) respiratory acidosis
Is able to determine that there is adequate
metabolic compensation

3) How will this patients ABG affect his management (especially with regards
to oxygen therapy)?
In terms of oxygen therapy:
o Try to give patient O2 via nasal prongs (1-2L/min) so that he can blow off
CO2.
o If known CO2 retainer, titrate oxygen to SaO2 88-92%, use of venturi
mask if necessary for precise oxygen delivery.
High flow O2 will compromise their respiratory drive. For the normal person,
breathing is driven by increased CO2 levels. In COPD patients however, their
respiratory centres have been dulled by chronic increased CO 2 levels therefore
hypoxia is driving their respiration. High flow O2 will dull this respiratory drive.

Grade Content Remarks


F Unable to answer
(30m)
P- Does not know that high flow O2 will compromise
(50m) their respiratory drive and lead to respiratory
depression
P Understands that high flow O2 will compromise
(70m) their respiratory drive and lead to respiratory
depression
P+ Understands that high flow O2 will compromise
(90m) their respiratory drive and lead to respiratory
depression
Suggests that the patient receive nasal prongs to
allow him to blow off the excess CO2.

4) What are the two major clinical syndromes that are classified as chronic
obstructive pulmonary disease? How do they differ? (Abbreviated question:
What is the pathophysiology of COPD ?)
Definition:
1. COPD is an intentionally imprecise term used to denote a process
characterised by the presence of chronic bronchitis or emphysema that
may lead to the development of fixed airway obstruction.
2. The hallmark of COPD is chronic inflammation that affects central airways,
peripheral airways, lung parenchyma and alveoli, and pulmonary
vasculature.

Page 32 of 67
3. The main components of these changes are narrowing and remodelling of
airways, increased number of goblet cells, enlargement of mucus-secreting
glands of the central airways, and, finally, subsequent vascular bed
changes leading to pulmonary hypertension.
Chronic bronchitis and emphysema are frequently encountered together in the
same patient.
Chronic bronchitis:
1. Clinical history of productive cough for 3 months of the year for 2
consecutive years.
2. Dyspnoea and airway obstruction, often with an element of reversibility,
are intermittently to continuously present.
3. Cigarette smoking is by far the leading cause of this disease, although
other inhaled irritants may induce the same process.
4. Pathologic events:
Inflammation in larger airways
Ciliary dysfunction
Mucosal thickening
Mucus hypersecretion- goblet cell size and number
N.B.: It is the inflammation in smaller bronchioles that is the
principal site of increased airflow obstruction.
Pulmonary emphysema:
1. Irreversible enlargement of the airspaces distal to the terminal bronchioles
2. Destruction of the walls of the terminal bronchioles, most often without
obvious fibrosis.
3. In the respiratory unit walls, there is elastin breakdown. This results in a
loss of appropriate recoil tension to support distal airways during
expiration.
4. Progressive dyspnoea and non-reversible obstruction accompany the
airspace destruction without mucus hypersecretion and productive
cough.
5. The loss of alveolar surface area and the accompanying capillary bed for
gas exchange contribute to the progressive hypoxia and dyspnoea.

Grade Content Remarks


F Loosely defines COPD
(30m)
P- Mentions that COPD can be classified into
(50m) chronic bronchitis or emphysema subtypes
P Mentions that COPD can be classified into
(70m) chronic bronchitisor emphysema subtypes
Is able to describe the pathological process
underlying both pathological subtypes
Understands that these are frequently
encountered together
P+ Items in P in great detail (especially microscopic
(90m) changes)

ABCs of ABG:
1. Look at pH: is it acidosis or alkylosis?
2. Does the amount of CO2 explain the pH?
a. Yes? Primary respiratory
b. No? Primary metabolic
3. If primary respiratory acidosis, for every in 10 of CO2, is HCO3 by 2, 4 or
something else?

Page 33 of 67
a. by 2mmHg acute resp. acid. with metabolic compensation
b. by 4mmHg chronic resp. acid. With metabolic compensation
c. None of the above mixed picture or no compensation
4. If primary respiratory alkylosis, for every in 10 of CO2, is HCO3 by 2.5?
a. Yes adequate metabolic compensation
b. No no compensation or mixed picture
5. If Primary metabolic acidosis or alkylosis, is the CO2 within 5mmHg of the last 2
digits of pH?
a. Yes adequate respiratory compensation
b. No mixed picture
Examples:

Metabolic acidosis and the Anion gap


Anion Gap = [K+] + [Na+] [Cl-] [HCO3-]
Normal value is 3 to 15
Anion Gap is a measure of ions not accounted for by the above formula
The blood needs to maintain electroneutrality (no. of +ive charges = no. of ive
charges). In normal conditions, if [HCO3-] is altered then [Cl-] will be altered to
maintain homeostasis. In the presence of other anions such as lactate, ketones
and other negatively charged proteins, the [Cl-] will not need to be altered to
maintain electroneutrality and so an increased anion gap results.

Page 34 of 67
Page 35 of 67
Pathology 2- Primary hyperparathyroidism

1) ASK FOR WHITE BOOK (spend at most 2mins on this segment): Test
examined: ______________________
Select one anatomical cut up the candidate observed. Please discuss the
pathological processes that contributed to the specimen you saw.
Select a morgue case.Have the candidate to describe the process of dissecting
the corpse.

Grade Content Remarks


F Has not prepared for the case at all
(30m)
P- Poorly describes the anatomical cut up or morgue
(50m) case
P For morgue:
(70m) o Detailed step-by-step explanation of how
the process of cutting up the patient
For anatomical cut up:
o Macroscopic and/or microscopic findings
P+ Opens with the history of the patient and how the
(90m) specimen was obtained
States the indication for the anatomical or
morgue cut up
For morgue:
o Detailed step-by-step explanation of how
the process of cutting up the patient
o Is able to relate these findings to the
pathology underpinning the disease
process
For anatomical cut up:
o Macroscopic and microscopic findings
o Is able to relate these findings to the
pathology underpinning the disease
process

Invite candidate to read the case history


A 56-year-old woman presents to her primary care physician complaining of progressive
fatigue, weakness, and diffuse bony pain. She says that her symptoms have been getting
worse over the past 2 months. Her medical history is notable for well-controlled
hypertension and recurrent renal stones. Physical examination is unremarkable. Blood
tests were performed. Here are the results:

Clinical biochemistry
Sodium (mmol/L) 138 135-
145
Potassium (mmol/L) 3.7 3.5-5.0
Chloride (mmol/L) 105 95-107
Calcium (mmol/L) 3.7 2.10-2.55
Phosphate (mmol/L) 0.4 0.8-1.5
25-hydroxy vitamin D (nmol/L) 120 50-
140
Alkaline phosphatase (U/L) 110 36-126

Page 36 of 67
Urea (mmol/L) 6.0 3.0-
8.0
Creatinine (mol/L) 87 60-110
Parathyroid hormone (pmol/L) 185 1.1-
6.9
A radionucleotide scan was also performed for this patient. The image is shown below:

1) What is your provisional diagnosis? What are your differential diagnoses for
hypercalcaemia?
PDx:
Primary hyperparathyroidism:
o Accounts for most cases of hypercalcemia
o Given the chronic nature of this womans Sx and the Hx of recurrent renal
stones, this is the most likely diagnosis.
Most likely due to a parathyroid adenoma or carcinoma (rare):
o Elevated PTH levels in presence of normal renal function and Vitamin D
DDx:
Primary hyperparathyroidism
Hyperthyroidism
Adrenal insufficiency
Medication-related hypercalcaemia
Granulomatous disease (TB or sarcoidosis)
Phaeochromocytoma
Medications: particularly lithium and thiazide diuretics
Grade Content Remarks
F Unable to offer a PDx or DDx
(30m)
P- Is able to get to the PDx with lots of prompting
(50m) from the examiner
P Mentions PDx and 2 DDx
(70m)
P+ Mentions PDx and 4 DDx
(90m)

2) How do you interpret the results from the clinical biochemistry?


Hyperparathyroidism
Hypercalcaemia
Hypophosphataemia

3) What are the complications of hyperparathyroidism?


Osteoperosis:
o PTH over-stimulation of bone resorption, with cortical bone affected >
cancellous bone bony fracture, especially of long bones
Hypercalcaemia:
o PTH also stimulates the kidneys to:

Page 37 of 67
Reabsorb calcium
Stimulates 25 (OH)-1- hydroxylase conversionof 25-
hydroxyvitamin D3 to its more active form of 1,25-dihydroxyvitamin
D3. This active vitamin D is responsible for the GI absorption of
calcium.
o Complications of hypercalcaemia (stones, bones, groans and psychiatric
overtones):
Stones- renal or biliary stones
Bones- Bone pain, osteitis fibrosa cystica
Groans- GI disturbance (LOA, nausea, constipation), acute
pancreatitis, peptic ulcers
Psychiatric overtones- Neuropsychiatric changes (e.g., mood
disturbance, fatigue, confusion, stupor, and coma)
Others: polyuria and polydipsia, short QT, bradycardia, AV block,
cardiac arrest

Grade Content Remarks


F Unable to answer even with much prompting
(30m) from the examiner
P- States 1-2 complications
(50m)
P States 4-5 complications
(70m)
P+ States 6 complications and is able to, in great
(90m) detail, describe the underlying pathological
processes.

4. What are the causes of primary hyperparathyroidism?


Multiple Endocrine Neoplasia (MEN):
o MEN type I (Wermer syndrome):
Autosomal dominant
Pancreatic islet cell tumour
Pituitary adenoma
Primary hyperparathyroidism
o MEN type IIa (Sipple syndrome):
Autosomal dominant
Phaeochromocytoma
Medullary carcinoma of the thyroid
Primary hyperparathyroidism
o MEN type IIb:
Same as IIa but no parathyroid involvement
Familial hypocalciuric hypercalaemia (FHH):
o Patients appear healthy.
o There may be a positive family history of other members with high serum
calcium without objective symptoms of nephrolithiasis or bone disease.
Malignancy:
o Occurs in patients with solid tumours of the lung, breast, kidney, ovary,
and head and neck; there is typically no bone metastasis.
o This process is mediated primarily by PTH-related-protein/peptide (PTHrP).
o PTHrP also plays a role in the hypercalcaemia associated with bone
metastases and multiple myeloma.
Multiple myeloma
Sarcoidosis
Excessive vitamin D intake

Page 38 of 67
Thyrotoxicosis
Chronic or acute leukaemia
Milk-alkali syndrome: patients usually have a history of excessive intake of
antacids
Thiazide diuretic
Grade Content Remarks
F Unable to answer even with much prompting
(30m) from the examiner
P- States 1-2 causes
(50m)
P States 4-5 causes
(70m)
P+ States 6 causes
(90m)

Pathology 3- Nephrotic syndrome

1) ASK FOR WHITE BOOK (spend at most 2mins on this segment): Test
examined: ______________________
Select one anatomical cut up the candidate observed. Please discuss the
pathological processes that contributed to the specimen you saw.
Select a morgue case.Have the candidate to describe the process of dissecting
the corpse.

Grade Content Remarks


F Has not prepared for the case at all
(30m)
P- Poorly describes the anatomical cut up or morgue
(50m) case
P For morgue:
(70m) o Detailed step-by-step explanation of how
the process of cutting up the patient
For anatomical cut up:
o Macroscopic and/or microscopic findings
P+ Opens with the history of the patient and how the
(90m) specimen was obtained
States the indication for the anatomical or
morgue cut up
For morgue:
o Detailed step-by-step explanation of how
the process of cutting up the patient
o Is able to relate these findings to the
pathology underpinning the disease
process
For anatomical cut up:
o Macroscopic and microscopic findings
o Is able to relate these findings to the
pathology underpinning the disease
process

Invite candidate to read the case history


A 48-year-old female has a 20-year history of type I diabetes mellitus. You had seen her
the previous day and arranged some urgent urine and blood tests. Her main complaint

Page 39 of 67
was ankle swelling extending up to her mid shin and, for the first few hours of the day,
some swelling around her eyes. She has also noticed foamy urine.

2) What is the likely diagnosis? How could you support this diagnosis?
PDx: Nephrotic syndrome:
o Facial and dependent oedema (peri-orbital oedema is the classical finding)
o Proteinuria (foamy urine)
o B/G of T1DM.
How would I support this diagnosis?
Serum albumin and cholesterol measurement.
Proteinuria:
o 24hr urine collection: The normal amount of protein is <150 mg/day.
o Patients excreting >3.5 g/day (or >3 g/day) are considered to have
nephrotic-range proteinuria.
o Total urine protein-to-creatinine ratio (mg/mg) on a random urine specimen
(first morning is preferred)
Normal urine protein-to-creatinine ratio is <0.2 mg/mg.
Ratios >3.5 mg/mg = nephrotic range for proteinuria.
Investigations:
o FBC, EUC, LFT (albumin), lipids, coags, BSL
o Urinalysis, MCS, PCR
o CXR, renal US

Grade Content Remarks


F Is not able to identify nephrotic syndrome
(30m)
P- Identifies nephrotic syndrome
(50m) Unable to suggest relevant investigations
P Identifies nephrotic syndrome
(70m) States the nephrotic range of protein
Suggest 1-2 relevant investigations
P+ Identifies nephrotic syndrome
(90m) States the nephrotic range of protein
Suggest 3-4 relevant investigations

3) Describe the main features and function of the glomerular filtration barrier.

The glomerular capillaries are lined by a fenestrated endothelium that sits on


the glomerular basement membrane, which in turn is covered by glomerular
epithelium, or podocytes, which envelops the capillaries with cellular extensions
called foot processes. In between the foot processes are the filtration slits

Page 40 of 67
made of nephrin. The podocytes have a negatively charged coat that repels
negatively charged stuff e.g. albumin.
Filtration of plasma water and solutes is extracellular and occurs through the
endothelial fenestrae and filtration slits. Cells, proteins and other macromolecules
are generally not able to pass through the glomerular filtration barrier.
ESL is the endothelial surface layer, which restricts plasma flow, can exclude
RBCs and some macromolecular solutes
Layers of the GBM:
o Lamina rara externa- adjacent to podocyte process, comprises heparin
sulphate and blocks by charge
o Lamina densa- dark central zone- comprises type 4 collagen & laminin &
blocks by size (MW>5800kDa)
o Lamina rara interna- adjacent to endothelial cells- comprises heparin
sulphate & blocks by charge
Grade Content Remarks
F Unable to answer question
(30m)
P- Mentions that it functions as a sift
(50m)
P Mentions several layers of the glomerular
(70m) filtration barrier:
o Fenestrated endothelium
o Glomerular basement membrane
o Podocytes
o Filtration slits made of nephrin
o ESL
P+ Mentions all layers of the glomerular filtration
(90m) barrier:
o Fenestrated endothelium
o Glomerular basement membrane
o Podocytes
o Filtration slits made of nephrin
o ESL
Understands that the podocytes have a
negatively charged
Layers of the GBM:
o Lamina rara externa
o Lamina densa
o Lamina rara interna

4) Blood tests reveal an albumin of 18g/L (normal 35-48g/L) and a total


cholesterol of 8.2mmol/L. Urine dipstick shows 4+ of protein.
How did this protein reach the urine and what are the main types of protein
likely to be present?
Selective proteinuria:
o Proteinuria that is >85% albumin is selective proteinuria.
o Albumin is negatively charged and the loss of glomerular membrane
negative charges could be important in causing albuminuria. Eg. in
minimal-change nephropathy proteinuria is selective.
Nonselective proteinuria:
o Glomerular leakage of all plasma proteins
o Would not involve changes in glomerular net charge but rather a
generalised defect in permeability.
Important proteins which are lost:

Page 41 of 67
o Antithrombin III
o Vitamin D binding protein,
o Immunoglobulin (in particular IgG)
o Transferrin
o Albumin
o Lipoprotein

Grade Content Remarks


F Unable to answer question
(30m)
P- Mentions either GBM damage or loss of
(50m) membrane negative charges
Mentions 1-2 important proteins which can be
lost
P Mentions that GBM damage + loss of membrane
(70m) negative charges
Mentions 3-4 important proteins which can be
lost
P+ Understands the difference between selective
(90m) and non-selective proteinuria
Mentions 5 important proteins which can be lost

5) Explain the pathophysiology of this womans oedema and her lipid changes.
What other complications can occur in this disease?
An increase in glomerular permeability albuminuria and hypoalbuminemia:
o In turn, hypoalbuminemia lowers the plasma colloid osmotic pressure,
causing greater transcapillary filtration of water throughout the body and
thus the development of oedema (Starlings equation)
In the nephrotic syndrome, levels of serum lipids are usually elevated.
o This can occur via:
Hypoproteinemia that stimulates protein, including lipoprotein,
synthesis by the liver.
Diminution of lipid catabolism caused by plasma levels of
lipoprotein lipase
Other complications
o DVT, PE : loss of antithrombin III and other anticoagulant proteins
o Immune deficiency: loss of Ig
o Malnutrition: catabolic state from protein losses
o Hypocalcaemia: loss of vit D binding protein
o CVD: hyperlipidaemia + thrombotic state

Grade Content Remarks


F Unable to answer question
(30m)
P- Mentions in glomerular permeability
(50m) albuminuria and hypoalbuminemia
Mentions at least 1 complication
P Mentions in glomerular permeability
(70m) albuminuria and hypoalbuminemia
Understand that this affects starling forces
oedema
Mentions at least 2 complications
P+ Mentions in glomerular permeability

Page 42 of 67
(90m) albuminuria and hypoalbuminemia
Understand that this affects starling forces
oedema
Mentions that hypoproteinemia protein
synthesis especially hepatic lipoprotein synthesis
hyperlipidaemia
Mentions at least 3 other complications

Page 43 of 67
Pathology 4- DKA
1) ASK FOR WHITE BOOK (spend at most 2mins on this segment): Test
examined: ______________________
Select one anatomical cut up the candidate observed. Please discuss the
pathological processes that contributed to the specimen you saw.
Select a morgue case.Have the candidate to describe the process of dissecting
the corpse.

Grade Content Remarks


F Has not prepared for the case at all
(30m)
P- Poorly describes the anatomical cut up or morgue
(50m) case
P For morgue:
(70m) o Detailed step-by-step explanation of how
the process of cutting up the patient
For anatomical cut up:
o Macroscopic and/or microscopic findings
P+ Opens with the history of the patient and how the
(90m) specimen was obtained
States the indication for the anatomical or
morgue cut up
For morgue:
o Detailed step-by-step explanation of how
the process of cutting up the patient
o Is able to relate these findings to the
pathology underpinning the disease
process
For anatomical cut up:
o Macroscopic and microscopic findings
o Is able to relate these findings to the
pathology underpinning the disease
process

Invite candidate to read the case history


A 13 year old girl was seen at home by her local doctor. She had been unwell for the past
week, with increasing weakness and lethargy, and had also complained of thirst and
vague abdominal pains. Her mother noted that the patient had not been eating, and had
vomited repeatedly over the past two days. Examination revealed a pale, thin, drowsy
girl with dry mucosae and sweet smelling breath. She was admitted to hospital.

Vital signs
Heart rate (b/min)
123
Blood pressure (mmHg)
106/67
Respiratory rate (/min) 32
Temperature (0C)
37.1

Page 44 of 67
1) How would you interpret the history and physical findings? What is the likely
diagnosis? What are your differential diagnoses?
PDx:
DKA- first presentation
DDx:
Hyperosmolar hyperglycaemic state (HHS)
Lactic acidosis
Starvation ketosis
Alcoholic ketoacidosis
Salicylate poisoning
Ethylene glycol/methanol intoxication
Uraemic acidosis
Grade Content Remarks
F No PDx or DDx offered
(30m)
P- Mentions only PDx
(50m)
P Mentions PDx and 1 DDx
(70m)
P+ Mentions PDx and 3 DDx
(90m)

2) What investigations would you order?


Bedside:
Use a cardiac monitor for continuous electrocardiographic monitoring to assess for
signs of:
o Hyperkalemia- peaked T-waves, widened QRS
o Hypokalemia- flattened or inverted T waves, ST depression, wide PR
interval
MSU:
o Ketones (dipstick)
o MCS
Finger prick:
o Capillary blood glucose
o Blood ketones (Optium meter)
Blood:
FBE
UECs + CMP
ABG/VBG (including bicarbonate)
Blood culture
For all newly diagnosed patients:
o Insulin antibodies
o Glutamic Acid Decarboxylase (GAD) antibodies (IAA and IA2 helps
differentiate between T1 and T2 DM)
o Coeliac screen (total IgA, anti-gliadin Ab, anti-endomycin, tissue
transglutaminase Ab)
o Thyroid function tests (TSH and FT4)

Interpret the following investigation findings:


ABG
pH: 6.50 7.35-7.45
PO2: 155 80-100
PCO2: 8.2 35-45

Page 45 of 67
HCO3: 1.9 24-34
Base access: -27.9 -3<BE<3

Metabolic acidosis with mixed picture: underlying respiratory alkylosis


Remember: if primary metabolic acidosis or alkylosis, is the CO 2 within 5mmHg of
the last 2 digits of pH?
o Yes adequate respiratory compensation
o No mixed picture

Grade Content Remarks


F No mention of relevant investigations
(30m) ABG:
Wrong interpretation of the ABG
P- Mentions non-appropriate investigations
(50m) ABG:
Wrong interpretation of the ABG
P Mentions a good variety of investigations but is
(70m) not able to organise them.
Provides the rationale for the proposed
investigations.
ABG:
Identifies metabolic acidosis
P+ Mentions a good variety of investigations
(90m) Is able to organise investigations into bedside,
blood and imaging studies, mentioning the most
important tests first.
ABG:
Identifies metabolic acidosis with mixed picture
Identifies possible underlying respiratory alkylosis

3) What is the pathophysiology of polyuria, polydipsia andketoacidosis in DKA?


T1DM: absolute insulin deficiency results from autoimmune destruction of -cells
in the pancreas.
Polyuria and polydipsia:
o Absolute insulin deficiency hyperglycaemiaoverwhelms the threshold
of the kidneys to reabsorb glucose glucosuria and an osmotic diuresis.
Ketoacidosis:
o Insulin deficiency in the presence of a relative excess of glucagon
breakdown of triglycerides into FFAs by activity of hormone sensitive
lipase.
o FFAs overwhelm the liver:
FFA are normally broken down into acetyl CoA then combined with
oxaloacetate to form citrate & enter the TCA cycle.
Instead they are diverted to ketone body production as
oxaloacetate is preferentially used for gluconeogenesis.
FFAs are converted to acetoacetate&-hydroxybutyric acid.
These ketone bodies are acidic, and consume HCO3- and cause a
metabolic acidosis.
o Kussmauls breathing occurs due to acidosis. Sweet breath occurs due to
carboxylation of acetoacetic acid to acetone.

Grade Content Remarks

Page 46 of 67
F Is not able to give an answer even after much
(30m) prompting
P- Brief description of the pathogenesis of polyuria
(50m) and polydipsia and ketoacidosis in DKA
P Recognises that there is absolute insulin
(70m) deficiency in T1DM
Describes in detail the pathophysiology of
polyuria and polydipsia
P+ Items in P PLUS:
(90m) Mentions the TCA cycle.
Identifies both ketone bodies responsible for
acidosis- acetoacetate & -hydroxybutyric acid

4) What other long term complications can develop? Describe the


pathophysiology underpinning these complications?
Macrovascular complications:
o risk of stroke, IHD and PVD
Microvascular complications:
o Diabetic retinopathy
o Diabetic nephropathy:
Glomerulosclerosis
Interstitial fibrosis
35% of patients develop ESRF
o Diabetic neuropathy:
Ischaemia from stenosis and occlusion of vasa nervorum.
Pathogenesis of the complications of DM:
1. Non-enzymatic glycosylation:
Hyperglycaemia formation of advanced glycosylation end-products (AGEs)
AGE formation on proteins such as collagen causes cross-links between
polypeptides:
o Trap LDL in blood vessel intima cholesterol depositionaccelerated
atherosclerosis risk of stroke, IHD and PVD
o Renal glomeruli: albumin binds to the glycated BM BM thickening
diabetic glomerulopathy.
2. Activation of protein kinase C:
Activation of intracellular protein kinase C:
o production of pro-angiogenic molecules (e.g. VEGF)
neovascularisation in diabetic retinopathy
o production of pro-fibrogenic molecules (e.g. TGF-) deposition of ECM
and BM material.
3. Intracellular hyperglycaemia with disturbances in polyol pathway:
Hyperglycaemia in tissues that dont usually require insulin for glucose transport
(e.g. nerves, lense, kidneys) intracellular glucose metabolised by aldose
reductase to sorbitol, a polyol.
Accumulation of sorbitol causes cell injury by:
o intracellular osmolality which causes water influx
o cell susceptibility to oxidative stress

Grade Content Remarks


F Brief description of the pathogenesis
(30m) Poor knowledge of the complications- mentions
1-2 complications
P- Mentions 3-4 complications

Page 47 of 67
(50m) Classifies them into macro- and micro-vascular
complications
P Mentions AGEs
(70m) Describes their effects on the endothelium and
basement membrane
P+ Items in P PLUS:
(90m) Describes how hyperglycaemia causes
disturbances in polyol pathway
Describes how the activation of intracellular
protein kinase C causes LT complications of T1DM

Page 48 of 67
Pathology 5 - Lymphoma
1) ASK FOR WHITE BOOK (spend at most 2mins on this segment): Test
examined: ______________________
Select one anatomical cut up the candidate observed. Please discuss the
pathological processes that contributed to the specimen you saw.
Select a morgue case.Have the candidate to describe the process of dissecting
the corpse.

Grade Content Remarks


F Has not prepared for the case at all
(30m)
P- Poorly describes the anatomical cut up or morgue
(50m) case
P For morgue:
(70m) o Detailed step-by-step explanation of how
the process of cutting up the patient
For anatomical cut up:
o Macroscopic and/or microscopic findings
P+ Opens with the history of the patient and how the
(90m) specimen was obtained
States the indication for the anatomical or
morgue cut up
For morgue:
o Detailed step-by-step explanation of how
the process of cutting up the patient
o Is able to relate these findings to the
pathology underpinning the disease
process
For anatomical cut up:
o Macroscopic and microscopic findings
o Is able to relate these findings to the
pathology underpinning the disease
process

Invite candidate to read the case history


A 62 year old man presents with a 4 week history of fever, sore throat and enlarged
cervical lymph nodes. He reports losing 5 kg over the period and also noted night sweats
during the last week.
Vital signs
Heart rate (b/min) 80
Blood pressure (mmHg)
125/85
Respiratory rate (/min) 18
Temperature (0C)
37.8
Examination revealed swelling of the oropharyngeal lymphoid tissues. Several lymph
nodes of 3 cm diameter were readily palpable in the anterior triangle of the neck and the
right axilla. No other lymph node groups were palpable. Spleen was mildly enlarged.

1) What are your differential diagnoses?


Differential:
Neoplastic

Page 49 of 67
o Primary lymphoma, leukaemia
o Secondary
Infective:
o Bacterial Strep, Staph, TB
o Viral EBV, HIV, CMV
o Protozoa - toxoplasmosis
o Fungal - histoplasmosis
Other:
o Autoimmune SLE, RA
o Drugs phenytoin
o Sarcoidosis
Grade Content Remarks
F No DDx offered
(30m)
P- Mentions 1-3 DDx
(50m)
P Mentions 4-5 DDx
(70m)
P+ Mentions 5 DDx
(90m) Is able to categorise DDx

2) What investigations would you order?


Laboratory:
FBC:
o WCC (infection) c.f. Pancytopaenia (lymphoma)
Blood Film:
o RS cell (Hodgkins lymphoma)
o Atypical lymphocytosis i.e. large irregular lymphocytes (EBV)
o Nucleated RBCs & giant platelets (suggests marrow involvement in
lymphoma)
Serology:
o EBV, CMV, HIV
Throat Swab MCS:
o Infection source
LFT: function in hepatomegaly
EUC: function may be compromised by LN compressing renal artery
Imaging:
PET-CT (first-line)
CXR: URTI, mediastinal mass in HL
LN biopsy (FNAB is preferred over excision biopsy. If confirmed stage 1 or 2, then
excision with radiotherapy is performed):
Microbiology (culture to exclude infectious causes)
Morphology (cytological and architectural abnormalities)
Immunohistochemistry (cell type)
Flow cytometry (cell type and quantification)
If these tests confirmed lymphoma, staging investigations could then be performed.
Staging investigations
LDH & ESR: = worse prognosis ( in response to cell damage, e.g. MI, lung
disease, lymphoma, anaemia, liver disease)
PET scan: involved sites appear hot as they take up FDG
CT: 2nd line staging investigation (look for mediastinal, liver, spleen, para-aortic
involvement)

Page 50 of 67
Bone marrow aspirate and trephine biopsy (bone marrow involvement, explain
paenias). Only if hot spot on PET.

Grade Content Remarks


F No mention of relevant investigations
(30m)
P- Mentions at least 2 appropriate investigations
(50m)
P Mentions a variety of investigations but is not
(70m) able to organise them.
Provides the rationale for the proposed
investigations.
P+ Items in P and:
(90m) Is able to organise investigations into laboratory,
imaging studies and special tests.

3) Biopsy of the cervical lymph node showed the following that patient had
diffuse large B-cell lymphoma. Describe a staging system used for lymphoma.
In this case, what is the stage?
The MODIFIED Ann Arbor classification" if you add the A,B,X,S,E symptoms
o Stage 1: Single lymph node group
o Stage 2: Multiple lymph node groups on same side of diaphragm
o Stage 3: Multiple lymph node groups on both sides of diaphragm (above
and below)
o Stage 4: Multiple extranodal sites or lymph nodes and extranodal disease
(e.g. liver and lung)
A = no systemic symptoms (or no B symptoms)
B = presence of symptoms (weight loss >10% in 6months, fever >38C,
drenching night sweats)
X = bulky disease (>1/3 widening of mediastinum at T5/T6, or >10 cm nodal
mass)
S = spleen involved
E = involvement of a single contiguous nodal site
This patient has oropharyngeal and cervical lymphoid tissue involvement and
spleen involvement he is at Stage IIIBof the disease

Grade Content Remarks


F Is unable to state any classification system

Page 51 of 67
(30m)
P- Vaguely describes the Ann Arbor classification
(50m) Unclear about the classification details
P Mentions the Ann Arbor classification
(70m) Provides the details of each stage
States the stage of the disease but not the
modifier (i.e. A, B, X, S and E)
P+ Items in P and:
(90m) Knows the modifiers to the Ann Arbor
classification (i.e. mentions the A, B, X, S and E)
Is able to accurately identify the stage and
modifier

4) What are some prognostic factors for this patient?


1) Patient factors:
a. Age
b. Performance status (ECOG)- ADL score from 0 to 5 (5- you are dead, 0- no
limitation), looks at ability to self-care and mobilise
2) Tumour factors:
a. Stage of disease (and bulk of disease)
b. No. of extranodal sites of disease
c. Serum LDH
d. If treatment has been commenced: 1) response to treatment and 2)
adverse effects of treatment
Grade Content Remarks
F No answer at all. Even after prompting
(30m)
P- 2 factors
(50m)
P At least 3 factors
(70m)
P+ Is able to categorise the factors (e.g. patient and
(90m) tumour factors)
Provides at least 3 factors

Page 52 of 67
Diagnostics
Diagnostics 1- Urinary tract infection

1) ASK FOR WHITE BOOK (spend only 2mins on this segment): Test examined:
_______________________
Ask about 1 lab test
Grade Content Remarks
F Has not prepared for the test at all
(30m)
P- Poorly knowledge of the test
(50m)
P States the indication for the test
(70m) Detailed step-by-step explanation of how the
process of processing the sample
P+ Opens with the history of the patient and how the
(90m) test specimen was obtained
States the indication for the test
Detailed step-by-step explanation of how the
process of processing the sample
Understands the clinical relevance/implications of
the results
Is able to relate findings back to the case
Is able to answer the examiners questions
regarding the tests

Invite candidate to read the case history


A 27-year-old, healthy, sexually active woman presents with pain on urination and recent
onset of urinary frequency and urgency. She has no costovertebral angle tenderness on
examination.

1) What are your provisional and differential diagnoses?


PDx: Urinary Tract Infection (irritant symptoms)
DDx:
o Non-infectious urethritis (from mechanical irritation)
o Over-active bladder (urgency, frequency, nil UTI)
o Vaginitis (vaginal discharge, itch, burning, due to candidiasis or
trichomonas)
o Urothelial carcinoma of the bladder or upper urinary tract

Grade Content Remarks


F No PDx or DDx offered
(30m)
P- Mentions only PDx
(50m)
P Mentions PDx and 1-2 DDx
(70m)
P+ Mentions PDx and 3 DDx
(90m)

2) What tests would you like to perform?

Page 53 of 67
Bedside:
Urine dipstick: nitrites ++, leukocytes ++
If +ve for nitrites and leukocytes then treat empirically. If ve then consider these:
Urine microscopy (UA): leukocytes and or bacteria, note: epithelial cells =
contamination
MSU:
o Urine Culture & Sensitivity: >105 colony forming units per mL or 108CFU/L
o Urine Gram stain: if +ve = 93% sensitive & 95% specific
gram ve rods = e. coli, klebsiella, enterobacteria, pseudomonas
gram +ve cocci = staphylococcus saphrophyticus
Blood:
EUC: check renal function
FBC
CRP
Blood cultures: if systemically unwell
Imaging:
Bladder Ultrasound: stone, tumour. Consider more in men, children, UTI not
responding to therapy, pyelonephritis or haematuria found
Renal US & KUB X-ray
Cystoscopy: if no answers still

Grade Content Remarks


F No mention of relevant investigations
(30m)
P- Mentions non-appropriate investigations
(50m)
P Mentions a good variety of investigations but is
(70m) not able to organise them.
Provides the rationale for the proposed
investigations.
P+ Items in P and:
(90m) Is able to organise investigations into bedside,
blood and imaging studies, mentioning the most
important tests first.

3) How would you do a MSU? Why is midstream important? What can we do


with urine after collection?
1. Wash hands & urethra + surrounding skin in normal soap
2. Begin urination (do not collect first urine passed may be contaminated with
skin/urethral
3. bacteria)
4. Collect urine midway through (need about 20-30mL in sterile screw top container)
which is
5. less likely to be contaminated
6. Tightly cap container
7. Wash hands
8. Label container
9. Take to lab as quickly as possible (reduce time allowed for further growth of
organisms)

Grade Content Remarks


F No idea how this is done
(30m)
P- Describes some of the above-mentioned steps

Page 54 of 67
(50m) (poor understanding of the process)
P Describes most of the above-mentioned steps
(70m)
P+ Describes all the above-mentioned steps
(90m) Mentions measures to avoid false positives when
the sample cannot be sent immediately to the
laboratory (e.g. freezing sample).

4a) A MSU specimen was collected from the patient. Interpret the results.
Would you treat this patient?

Allow the candidates to recognise that this is a contaminated specimen. If not prompt as
required
MSU 1 - Contaminated specimen
Result Units Ref Interval
Urine White Blood Cells >100 x10E6/L (<10)
Urine Red Blood Cells <10 x10E6/L (<10)
Urine Squamous Epithelial cells >100 x10E6/L (<10)
Organism 1 (Gram -ve Rods) >100 x10E6/L
Organism 2 (Gram +ve Coccus) >100 x10E6/L
Upon recognition of contaminated specimen,

4b) A second MSU was collected from this patient. Interpret the results. Would
you treat this patient? What is the causative organism in this case?

MSU 2 - E.Coli UTI


Result Units Ref Interval
Urine White Blood Cells >100 x10E6/L (<10)
Urine Red Blood Cells <10 x10E6/L (<10)
Urine Squamous Epithelial cells <10 x10E6/L (<10)
Organism 1 (Gram -ve Rods) >100 x10E6/L

4c) What is a positive result for UTI?


One organism: >108
If two organisms: only 1 can be >108
If two of them are >108 then it is likely that the sample is contaminated
Polymorphs: >108
Increased squamous cell: contamination

Grade Content Remarks


F Is not able to answer
(30m)
P- Confused between 108 CFU/L and 105 CFU/mL
(50m) Knows that there is a UTI but unable to tell that
specimen is contaminated

Page 55 of 67
P Able to identify contaminated specimen and
(70m) specimen with UTI but unable to identify e.coli
Understands that one organism needs to be >108
for UTI to be diagnosed
Understands that if there are two organisms from
the culture, only 1 can be >108
P+ Correctly identifies contaminated specimen and
(90m) specimen with UTI. Able to identify e.coli as
causative organism.
Understands that if there are two organisms from
the culture, only 1 can be >108
Knows when the sample is contaminated

5) What organisms can cause a UTI and what organisms can contaminate the
MSU?
Common UTI organisms:
1. E. coli = 70-95% (gram ve bacillus aerobic and facultative anaerobic)
2. Staph saprophyticus = 5-20% (gram +ve cocci facultative anaerobe)
3. Proteus mirabilis = ~1% (gram ve facultative anaerobe)
4. Klebsiella (men, children)
5. Pseudomonas aeruginosa (instituitionalised/IDUC)
6. Enterococci
7.
Common organisms that can contaminate the MSU are from the normal flora of the
periurethral and urethral regions:
Lactobacilli
Coagulase-negative staphylococci
Corynebacteria
Streptococci

Grade Content Remarks


F Lists 1-2 common UTI organisms
(30m)
P- Lists 3-4 common UTI organisms
(50m)
P Lists at least 5 common UTI organisms
(70m)
P+ Lists at least 5 common UTI organisms
(90m) Lists at least 2 organisms that can contaminate
the MSU

6) How will you treat this womans UTI?


Pharmacological:
Trimethoprim 300 mg orally, daily for 3 days OR
Cephalexin 500 mg orally, 12-hourly for 5 days OR
Amoxycillin+clavulanate 500+125 mg orally, 12-hourly for 5 days OR
Nitrofurantoin 100 mg orally, 12-hourly for 5 days.
Because this woman is 27yo and can become pregnant, avoid
trimethoprim or trimethoprim + sulfamethoxazole (Bactrim)
For severe infection:
Gentamicin IV PLUS amoxy/ampicillin 2 g IV, 6-hourly.
Non-pharmacological:
Drink plenty of water to urinate frequently assists flushing

Page 56 of 67
Grade Content Remarks
F Lists 1-2 antibiotics
(30m)
P- Lists 3-4 antibiotics
(50m)
P Lists 3-4 antibiotics
(70m) Understands how severe infection can be treated
Mentions pharmacological and non-pharmacological
methods of treatment
P+ Lists 3-4 antibiotics
(90m) Understands how severe infection can be treated
Appreciates the potential that this woman may become
pregnant and alters management accordingly
Mentions pharmacological and non-pharmacological
methods of treatment

7) How would we test for sensitivity?


The CDS system is used. CDS = Calibration-Dichotomous-Susceptibility
o Organism is cultured on agar plates containing discs impregnated with
different antibiotics.
o A zone of inhibition is seen around those antibiotics the micro-organism
is sensitive to
Zone of inhibition = area where no colonies are seen around the
antibiotic impregnated disc. 6mm annulus radius for most
antibiotics, 2mm if vancomycin
The disc itself is 6mm too, so basically a zone with an 18mm
diameter must be found to say the micro-organism is susceptible.
o Note: these values can vary based on antibiotic diffusion constant.
o Results are graded as follows in the CDS system:
Susceptible
Resistant

Page 57 of 67
Grade Content Remarks
F Is unclear how this is performed
(30m)
P- Very brief description of CDS
(50m)
P Describes the above-mentioned steps (in the
(70m) picture)
States the types of antibiotic discs used
P+ Describes the above-mentioned steps (in the
(90m) picture)
States the types of antibiotic discs used
Mentions the need to measure the zone of
inhibition and that this is smaller for vancomycin

Page 58 of 67
Diagnostics 2- meningitis

1) ASK FOR WHITE BOOK (spend at most2mins on this segment): Test


examined: _______________________
Ask about 1 lab test

Grade Content Remarks


F Has not prepared for the test at all
(30m)
P- Poorly knowledge of the test
(50m)
P States the indication for the test
(70m) Detailed step-by-step explanation of how the
process of processing the sample
P+ Opens with the history of the patient and how the
(90m) test specimen was obtained
States the indication for the test
Detailed step-by-step explanation of how the
process of processing the sample
Understands the clinical relevance/implications of
the results
Is able to relate findings back to the case
Is able to answer the examiners questions
regarding the tests

Invite candidate to read the case history


A 25-year-old man presents to the emergency department with fever and in a confused,
irrational state. He is accompanied by his wife, who provides the history. She states that
he had been well until approximately 1 week ago, he had a nose operation. On the
morning of admission, he complained of progressive severe headache and nausea. He
vomited once.
On examination, He is lethargic, confused, lying with his hand over his eyes. Fundoscopic
examination shows no papilledema and there is a petechial rash present:

Vital signs
Heart rate (b/min)
100
Blood pressure (mmHg)
95/60
Respiratory rate (/min) 18
Temperature (0C)
39.0

2) What is your provisional diagnosis? What infectious diagnosis is suggested?


What are the most likely etiologic agents in this patient? Or a newborn?
PDx: Meningitis secondary to Neisseria meningitides
What infectious diagnosis is suggested?
The acuity and severity of presentation are most consistent with a pyogenic
bacterial cause, although viral, mycobacterial, and fungal causes should be
considered as well.
In adults:

Page 59 of 67
o Neisseria meningitidis
o S pneumoniae
o Haemophillus influenza type B (if not immunised)
Newborns <3 months (the most common pathogens are those to which the infant
is exposed in the maternal genitourinary canal) remember GEL:
o GBS
o E coli
o Listeria monocytogenes
o Other gram-negative bacilli
Grade Content Remarks
F Is able to get the PDx but not the aetiological agents
(30m)
P- Mentions PDx
(50m) Is able to give some aetiological agents
P Mentions PDx
(70m) Is able to give most of the abovementioned aetiological
agents basedon age
P+ Mentions PDx
(90m) Mention that meningitis can be cause by fungal and viral
infections
Is able to give most of the abovementioned aetiological
agents based on age
Mention that the patient most likely has a bacterial
meningitis

3) What tests should be performed to confirm the diagnosis?


Bedside tests:
Capillary blood glucose: check in shock
Throat swabs (1 for bacteria, 1 for virology)
Urine MCS: for e.coli, meningitis
Stool sample: enteroviruses
Bloods:
Blood culture:
o +ve for S. pneumoniae
o ve in 30% of meningococcal infection
CRP: good to rule OUT (97% NPV) bacterial meningitis if ve. Could then be viral.
Virology PCR of CSF herpes virus
FBC: neutrophilia
EUC: renal impairment if shocked
LFT: baseline
Coagulation screen: intracranial haemorrhage risk, DIC secondary to sepsis
causing endothelial cell injury
Special tests:
Emergent lumbar puncture:
o Contraindications:
Any signs of raised ICP
Bleeding diathesis
Infection at the site of LP
o If there is focal neurological deficit, papilloedema, new seizure, history of
CNS disease or intracranial mass lesion, because of coning)CT or MRI of
the brain may be performed before the LP.
o Give Abx first-time to Abx
What you should do with the CSF:

Page 60 of 67
o Opening pressure: 7-18cm = N, >40 = likely meningitis, 14-30 = typical in
meningitis
o Appearance
o CSF gram stain: +ve for causative organisms in 50-90% (bacterial), -ve
(viral)
o CSF cell count: turbid, neutrophils ++ (bacterial), lymphocytes ++ (viral).
o CSF protein: elevated (bacterial/viral)
o CSF glucose: low (bacterial/viral use glucose for energy)
o CSF culture: positive (bacterial), -ve (viral)
o Antigen detection for CSF: N. meningitidis capsular LPS antigen

Imaging:
CXR: pneumonia (s. pneumoniae is a common cause of meningitis)
Grade Content Remarks
F No mention of relevant investigations
(30m)
P- Mentions non-appropriate investigations
(50m)
P Mentions a good variety of investigations but is
(70m) not able to organise them.
Provides the rationale for the proposed
investigations.
P+ Items in P and:
(90m) Is able to organise investigations into bedside,
blood and imaging studies, mentioning the most
important tests first.
Volunteers further tests for LP
Knows contraindications for LP

4) What kinds of treatments should be started or considered? Why?


Empirical therapy: Dexamethasone 10 mg IV, starting before or with the
first dose of antibiotic, then 6-hourly for 4 days
PLUSceftriaxone 4 g IV daily OR ceftriaxone 2g IV 12
hourly OR cefotaxime 2g IV 6-hourly
If no pathogen is isolated, continue the empirical
antibiotics for a minimum of 10 days, depending on
response. Dexamethasone should be continued for a
total duration of 4 days.
If gram-positive Add vancomycin 1.5 g IV, 12-hourly
diplococci are seen or a This is to ensure that Streptococcus pneumoniae
pneumococcal antigen isolates that display resistance to penicillin and/or
assay in CSF is positive cephalosporins are adequately covered.
If adults >50 or Add Benzyl penicillin 2.4g IV, 4-hourly
immunocompromised, cover
Listeria Monocytogenes
Prophylaxis is essential Ceftriaxone 250 mg IM, as a single dose (preferred
for close contacts: for pregnant women) OR Ceftriaxone 500mg Orally
OR Rifampicin 600mg (children 10mg/kg).

Directed therapy for meningitis due to Neisseria meningitides:


o Benzylpenicillin 1.8 g IV, 4-hourly for 3 to 5 days.
Empirical therapy for patients with penicillin hypersensitivity:

Page 61 of 67
o Vancomycin 1.5grams IV 12-hourly AND ciprofloxacin 400mg IV, 12-hourly.
Note Vanc: adjust for renal function, monitor blood concentrations
o OR Moxifloxacin 400mg IV, daily
N.B.:
Antibiotics should be started immediately, without waiting for imaging study or
lumbar puncture if delay is anticipated in these procedures.
The use of corticosteroids has been shown to decrease the risk of sensorineural
hearing loss among children with H influenzae meningitis and mortality among
adults with pneumococcal meningitis.
The benefit of adjuvant corticosteroids for other types of meningitis is unproven.

Grade Content Remarks


F Is not able to recall the empirical therapy
(30m)
P- Mentions the empirical therapy:
(50m) o Dexamethasone PLUS
o Ceftriaxone
P Knows not to delay antibiotic therapy
(70m) Mentions the empirical therapy:
o Dexamethasone PLUS
o Ceftriaxone
P+ Knows not to delay antibiotic therapy
(90m) Mentions the empirical therapy:
o Dexamethasone PLUS
o Ceftriaxone
Knows to add vancomycin when patient has
pneumococcal infection
Knows to add Benzyl penicillin when patient has
Listeria Monocytogenes infection

5) What is the pathogenesis of this mans presentation? What factors aid the
colonisation and spread of the bacteria?
Colonisation of the hosts nasopharynx local invasion of the mucosal epithelium
bacteraemia cerebral endothelial cell injury follows BBB permeability,
facilitating meningeal invasion.
The resultant inflammatory response in the subarachnoid space causes cerebral
oedema, vasculitis, and infarction CSF, hydrocephalus, worsening cerebral
oedema, ICP, and cerebral blood flow.

Factors that aid colonisation and spread of bacteria:


Nasal colonisation is facilitated by pili on the bacterial surface of N
meningitidis that assist in mucosal attachment.
N meningitidis, H influenzae, and S pneumoniae also produce IgA proteases that
cleave IgA, the antibody commonly responsible for inhibiting adherence of
pathogens to the mucosal surface.
By cleaving the antibody, the bacteria are able to evade this important host
defence mechanism.
In addition, N meningitidis, H influenzae, and S pneumoniae are often
encapsulated, which can assist in nasopharyngeal colonisation as well as
systemic invasion. The capsule inhibits neutrophil phagocytosis and resists classic

Page 62 of 67
complement-mediated bactericidal activity, enhancing bacterial survival and
replication.
Grade Content Remarks
F Offers no answers
(30m)
P- Briefly describes the pathogenesis of N.
(50m) meningitides meningitis
P Is able to in great detail describe the
(70m) pathogenesis of N. meningitides meningitis
P+ Is able to in great detail describe the
(90m) pathogenesis of N. meningitides meningitis
Is able to state 1-2 virulence factors

Page 63 of 67
Diagnostics 3- Infective endocarditis
A 55-year-old man who recently emigrated from China presents to the emergency
department with fever. He has had recurring fevers over the past 3 weeks, associated
with chills, night sweats, and malaise. He admits to infrequent intravenous heroin use
and has a 10-pack-year smoking history.
Ophthalmoscopic examination is remarkable for retinal haemorrhages. Cardiac
examination is notable for a grade 3/6 pansystolic murmur heard loudest at the left lower
sternal border.

Vital signs
Heart rate (b/min)
108
Blood pressure (mmHg)
120/80
Respiratory rate (/min) 16
Temperature (0C)
38.5
Oxygen saturations
97% on RA

1) What is your provisional diagnosis and differentialdiagnoses?


PDx:
Infective Endocarditis- infection of the endocardial surface of heart (valvular
surfaces, chordae tendinae, mural endocardium). Fever + new murmur = IE
until proven otherwise
DDx:
Rheumatic fever
LibmanSacks endocarditis:
o A form of nonbacterial endocarditis that is seen in SLE.
o It is one of the most common cardiac manifestations of lupus (the most
common being pericarditis)
Sepsis
Bleeding disorder
Atrial myxoma
Non-bacterial thrombotic endocarditis (NBTE):
o Most common tumours producing NBTE are pancreatic, lung, and colon, so
patients may report signs and symptoms consistent with those underlying
conditions.
o Also may report symptoms of underlying hypercoagulable state such as
recurrent DVT, accelerated atherosclerosis, pre-mature CAD, MI, or stroke.

Grade Content Remarks


F No PDx or DDx offered
(30m)
P- Mentions only PDx
(50m)
P Mentions PDx and 1 DDx
(70m)
P+ Mentions PDx and 3 DDx
(90m)

2) What investigations are appropriate?

Page 64 of 67
Bloods:
FBC: leucocytosis and neutrophilia, normocytic anaemia
Blood Cultures: +ve in 2/3 sets
ECG:
o Can have conduction problems e.g. prolonged PR
o New AV block is suggestive of abscess formation
ESR: non-specific but indicates inflammation rather than bleeding disorder
Imaging:
Echocardiogram:
o Valvular vegetations
CXR:
o Cardiomegaly
o CXR: evidence of heart failure and multiple pulmonary infiltrates (in right-
sided endocarditis)
Grade Content Remarks
F No mention of relevant investigations
(30m)
P- Mentions non-appropriate investigations
(50m)
P Mentions a good variety of investigations but is
(70m) not able to organise them.
Provides rationale for the proposed
investigations.
P+ Items in P and:
(90m) Is able to organise investigations into bedside,
blood and imaging studies, mentioning the most
important tests first (blood cultures and echo).

3) Describe how you would collect the blood cultures


Blood cultures are taken BEFORE antibiotics are given. If empirical antibiotics
must be given emergently then take blood cultures as soon as possible
afterwards.
3 blood cultures should be taken within a 24 hour period ideally about 1hr apart
o Each time, about 10mL is put in an aerobic tube and ~10mL in an
anaerobic tube
o Aerobic before the anaerobic bottle.
Take from peripheral rather than central or femoral sites to reduce false +ves
Use strict sterile technique to avoid contaminating results with skin flora (use a
chlorhexidine swab, not alcohol swab):
o 5cm area disinfected with chlorhexidine in concentrically enlarging circles
around site
o Can also sterilise the blood culture cap with 70% alcohol
o This is important to correctly identify the causative organism so
appropriateantibiotics can be given
Pyridoxal needs to be added to the blood culture media to grow some streptococci
If 2 blood cultures are +ve for the same micro-organism then it is very likely there
is abacteraemia caused by that organism
Summary: Antibiotics, 3 cultures, sterile technique, 2/3 for positive
Grade Content Remarks
F No idea how this is performed
(30m)
P- Describes some of the above-mentioned steps
(50m) (poor understanding of the process)

Page 65 of 67
P Describes most of the above-mentioned steps
(70m)
P+ Describes all the above-mentioned steps
(90m) Mentions that blood cultures should be taken
BEFORE antibiotics are given but in situations
when empirical antibiotics must be given
emergently, blood cultures can be taken ASAP
afterwards.

4) What is the diagnostic criterion for infective endocarditis?


Duke criteria (must meet 2 major criteria or 1 major and 3 minor criteria or 5
minor criteria):
Major criteria:
o Positive blood culture for infective endocarditis (IE):
Typical micro-organism for IE from 2 separate blood cultures
Persistently positive blood cultures.
o Evidence of endocardial involvement:
Oscillating intracardiac mass on valve/supporting structures, or in
the path of regurgitant jet in the absence of an alternative
anatomical explanation
Abscess
New partial dehiscence of prosthetic valve or new valvular
regurgitation.
Minor criteria:
o Predisposing heart condition or intravenous drug use
o Fever over 38C
o Vascular phenomenon such as major arterial emboli, septic pulmonary
infarcts, mycotic aneurysm, intracranial haemorrhage, conjunctival
haemorrhage, Janeway lesions
o Immunological phenomenon:
Glomerulonephritis
Osler nodes
Roth spots
Rheumatoid factor
o Microbiological evidence:
Positive blood cultures not meeting major criteria
o Echocardiogram:
Consistent with IE but not meeting major criterion.
Grade Content Remarks
F Offers no answer
(30m)
P- Mentions the Duke criteria without knowing what
(50m) constitutes it.
P Mentions that the Duke criteria consists of:
(70m) o Major criteria
o Minor criteria
o Must meet 2 major criteria or 1 major and 3
minor criteria or 5 minor criteria
P+ Mentions that the Duke criteria consists of:
(90m) o Major criteria
o Minor criteria
Must meet 2 major criteria or 1 major and 3 minor
criteria or 5 minor criteria
Is able to give 2-3 examples of items in the major and

Page 66 of 67
minor criteria

5) What are the common causative agents?If the patient is not responding to
initial antibiotic therapy, what organisms should you be considering?
IVDU (SSEG): staph aureus, streptococci, enterococci, gram ve bacilli
Native valve (SVC): staph aureus, viridians (oral) group streptococci (strep
sanguis, oralis), coagulase negative staphylococci
Prosthetic valve endocarditis (SSE): staph aureus, staph epidermidis, enterococci
Viridans Group Streptococcus (s. sanguis, oralis, salivarius) = commensal flora of
the mouth and GIT
o Gram +ve cocci, coagulase negative, alpha (green) haemolytic
o Typically produce a greenish (viridians) discoloration on blood agar
o Leading cause of subacute bacterial endocarditis (SBE)
If the patient is not responding to initial antibiotic therapy, consider the HACEK group of
organisms:
Usually culture-negative
Haemophilus species
Actinobacillus actinomycetemcomitans
Cardiobacterium hominis
Eikenella corrodens
Kingella

Grade Content Remarks


F Offers no answer
(30m)
P- Mentions 1-2 common causative agents
(50m)
P Mentions 3-4 common causative agents
(70m) Mention that S. Aureus is the most common
cause
Mentions 1-2 HACEK organisms
P+ Mentions 3-4 common causative agents
(90m) Mention that S. Aureus is the most common
cause
Mentions ALL HACEK organisms

Page 67 of 67

Potrebbero piacerti anche